K
Khách

Hãy nhập câu hỏi của bạn vào đây, nếu là tài khoản VIP, bạn sẽ được ưu tiên trả lời.

a: BC=căn 3^2+4^2=5cm

AB<AC<BC

=>góc C<góc B<góc A

b: Xét tứ giác ABCD có

M là trung điểm chung của AC và BD

=>ABCD là hình bình hành

=>AB//CD

=>CD vuông góc CA

c: CM=1/2CA=2cm

Xét ΔCBD có

CM,BN là trung tuyến

CM cắt BN tại H

=>H là trọng tâm

=>CH=2/3CM=2/3*2=4/3(cm)

d: Xét ΔDBC có

DKlà trung tuyến

H là trọng tâm

=>D,K,H thẳng hàng

5:

a: ΔABC cân tại A

mà AH là trung tuyến

nên AH vuông góc BC

BH=CH=4cm

=>AH=căn 10^2-4^2=2*căn 21(cm)

b: Xét ΔIBH và ΔIAD có

góc IBH=góc IAD

IB=IA

góc BIH=góc AID

=>ΔIBH=ΔIAD

=>AD=BH=HC

 

20 tháng 5 2022

Ap dụng định lý py ta go ta có 
\(BC^2=AB^2+AC^2\\ BC^2=9+16=25\\ BC=5\left(cm\right)\)
 

a: AC=căn 5^2-3^2=4cm

b: Xét ΔMAB và ΔMCD có

MA=MC

góc AMB=góc CMD

MB=MD

=>ΔMAB=ΔMCD

=>AB=CD

c: AB+BC=CD+BC>DB=2BM(ĐPCM)

24 tháng 3 2022

A) Vì tam giác ABC vuông tại A nên ta có :

      AB2+AC2=BC2AB2+AC2=BC2

⇔AC2=BC2−AB2⇔AC2=BC2−AB2

⇔AC2=52−32⇔AC2=52−32

⇔AC2=25−9⇔AC2=25−9

⇔AC2=16⇔AC2=16

⇔AC=4

 

a: \(BC=\sqrt{6^2+3^2}=3\sqrt{5}\left(cm\right)\)

\(BM=\sqrt{6^2+1.5^2}=\dfrac{3\sqrt{17}}{2}\left(cm\right)\)

b: Xét tứ giác ABCD có 
M là trung điểm của BD

M là trung điểm của AC

Do đó: ABCD là hình bình hành

Suy ra: AB=CD và CD//AB

hay CD\(\perp\)AC

10 tháng 5 2022

Không làm thì CÚT

29 tháng 3 2021

xét ΔABM và ΔCDM :

         AM = CM ( M là t/đ của AC )

       góc AMB = góc CMD ( đối đỉnh )

      MB = MD ( gt)

do đó : ΔABM = ΔCDM ( c.g.c )

b) Ta có: ΔABM=ΔCDM(cmt)

nên \(\widehat{MAB}=\widehat{MCD}\)(hai góc tương ứng)

mà \(\widehat{MAB}=90^0\)(gt)

nên \(\widehat{MCD}=90^0\)

Ta có: \(\widehat{MCD}+\widehat{MCB}=\widehat{DCB}\)(Tia CM nằm giữa hai tia CD,CB)

nên \(\widehat{DCB}>\widehat{MCD}\)

hay \(\widehat{DCB}>90^0\)

Xét ΔDCB có \(\widehat{DCB}>90^0\)(cmt)

mà cạnh đối diện với \(\widehat{DCB}\) là cạnh DB

nên DB là cạnh lớn nhất trong ΔDCB(Định lí)

hay DB>BC

mà BC>AC(ΔABC vuông tại A có BC là cạnh huyền nên BC là cạnh lớn nhất)

nên AC<BD(Đpcm)

a) Áp dụng định lí Pytago vào ΔABC vuông tại A, ta được:

\(BC^2=AB^2+AC^2\)

\(\Leftrightarrow AC^2=BC^2-AB^2=5^2-3^2=16\)

hay AC=4(cm)

Vậy: AC=4cm